Proving the triple curl identity

In summary, the homework equation states that \nabla \times (\nabla \times \vec{A}) = \nabla(\nabla \cdot \vec{A}) - (\nabla \cdot \nabla)\vec{A}
  • #1
anonymous188
17
0

Homework Statement



Prove that [tex]\nabla \times (\nabla \times \vec{A}) = \nabla(\nabla \cdot \vec{A}) - (\nabla \cdot \nabla)\vec{A}[/tex] using Einstein notation.

Homework Equations



[tex]\nabla \times (\nabla \times \vec{A}) = \nabla(\nabla \cdot \vec{A}) - (\nabla \cdot \nabla)\vec{A}[/tex]

[tex] \varepsilon_{ijk}\varepsilon_{imn} = \delta_{jm}\delta_{kn} - \delta_{jn}\delta_{km}[/tex]

The Attempt at a Solution



[tex] \begin{equation*}
\begin{split}
\nabla \times (\nabla \times \vec{A})& = \nabla \times (\varepsilon_{ijk}\frac{\partial A_k}{\partial_j})_i\\
& = [\varepsilon_{lmn} \frac{\partial}{\partial_m}(\varepsilon_{njk}\frac{\partial A_k}{\partial_j})_n]_l\\
& = [-\varepsilon_{nml}\varepsilon_{njk} \frac{\partial}{\partial_m}(\frac{\partial A_k}{\partial_j})_n]_l\\\
& =[(\delta_{mk}\delta_{lj} - \delta_{jm}\delta_{lk})\frac{\partial}{\partial_m}(\varepsilon_{ijk}\frac{\partial A_k}{\partial_j})_n]_l\\
& = [\frac{\partial}{\partial_k}(\frac{\partial A_k}{\partial_j})_n - \frac{\partial}{\partial_m}(\frac{\partial A_k}{\partial_m})_n]_l
\end{split}
\end{equation}
[/tex]

One step that I'm not sure about was pulling [itex]\epsilon_{njk}[/itex] out in step 3. Although it's a constant, it seems like it would depend on [itex]n[/itex], so I don't know if I'm allowed to do that. If everything is right up to the last step, then I'm confused about what to do next. What I have in the last line definitely doesn't look close to the answer.

Thanks in advance for any help with this problem. If you could just provide hints as to the next step or errors in my logic rather than giving away the solution, that would be great. I usually don't like asking for help, but I'm in a difficult situation at the moment.

Thanks!
 
Last edited:
Physics news on Phys.org
  • #2
Hmm.. I made a few typographical errors, but it seems my editing hasn't changed anything. Maybe it takes a while to update.
 
  • #3
Your notation [itex]\partial/\partial j[/itex] is confusing. I would suggest either [itex]\partial/\partial x_j[/itex] or [itex]\partial_j[/itex].

Anyway, you're very close. Just try writing out the RHS of the original equation in component form.
 
  • #4
I wonder if I'm allowed to switch the differential operators in the first term of the last line. And then, since the second term is a constant, maybe I can pull it out of the vector. Doing that,

[tex] \begin{equation*}
\begin{split}
\nabla \times (\nabla \times \vec{A})& = \nabla \times (\varepsilon_{ijk}\frac{\partial A_k}{\partial_j})_i\\
& = [\varepsilon_{lmn} \frac{\partial}{\partial_m}(\varepsilon_{njk}\frac{\partial A_k}{\partial_j})_n]_l\\
& = [-\varepsilon_{nml}\varepsilon_{njk} \frac{\partial}{\partial_m}(\frac{\partial A_k}{\partial_j})_n]_l\\\
& =[(\delta_{mk}\delta_{lj} - \delta_{jm}\delta_{lk})\frac{\partial}{\partial_m}(\varepsilon_{ijk}\frac{\partial A_k}{\partial_j})_n]_l\\
& = [\frac{\partial}{\partial_k}(\frac{\partial A_k}{\partial_j})_n - \frac{\partial}{\partial_m}(\frac{\partial A_k}{\partial_m})_n]_l\\
& = [\frac{\partial}{\partial_j}(\frac{\partial A_k}{\partial_k})_n]_l - \frac{\partial^2 A_k}{\partial^2_m}
\end{split}
\end{equation}
[/tex]

Now it looks closer to the answer, I think. The only problem is that the first derivative of the first term should have the same index as the the component of the vector, i.e. [itex]n[/itex] should be [itex]j[/itex]. And it seems like the second term isn't a constant since it also has that subscript [itex]n[/itex]. Hence my confusion.
 
  • #5
You are using the partial derivative notation a bit funny. I would write your final expression as
[tex]\partial_j \partial_k A_k-\partial_m \partial_m A_k[/tex]. And that's sort of what you want. I've dropped the funny 'component' subscripts, you don't need them, e.g.
[tex](\varepsilon_{ijk} \partial_j A_k)[/tex] is the ith component of a vector. I can tell that because j and k both occur twice, which makes them 'dummy' indices. Which is what's wrong with your final expression. In one term k is a dummy variable, in the other it's not. Can you figure out how that happened? Keep track of what is 'dummy' in any given step and make sure it stays that way.
 
Last edited:

Related to Proving the triple curl identity

1. What is the triple curl identity and why is it important?

The triple curl identity is a mathematical equation that relates the curl of a vector field to the divergence of the curl of that same vector field. It is important because it is a fundamental concept in vector calculus and has many applications in physics, engineering, and other scientific fields.

2. How do you prove the triple curl identity?

The triple curl identity can be proven using vector calculus and properties of the curl and divergence operators. It involves manipulating the equation and using the properties of these operators to show that the two sides of the identity are equal.

3. What are the applications of the triple curl identity?

The triple curl identity has many applications in physics, engineering, and other scientific fields. It can be used to solve various problems involving electromagnetic fields, fluid flow, and other vector fields. It is also used in the derivation of other important equations, such as the Maxwell's equations.

4. Is the triple curl identity always true?

Yes, the triple curl identity is always true. It is a fundamental mathematical concept and holds for any vector field in any dimension. However, it may not always be easy to prove or apply in certain situations.

5. How can understanding the triple curl identity benefit my scientific research?

Understanding the triple curl identity can benefit your scientific research by providing you with a powerful tool for solving problems involving vector fields. It can also help you gain a deeper understanding of the relationships between different physical quantities and improve your mathematical skills in vector calculus.

Similar threads

  • Calculus and Beyond Homework Help
Replies
3
Views
1K
  • Calculus and Beyond Homework Help
Replies
3
Views
1K
  • Cosmology
Replies
9
Views
1K
  • Calculus and Beyond Homework Help
Replies
2
Views
2K
  • Calculus and Beyond Homework Help
Replies
3
Views
2K
  • Calculus and Beyond Homework Help
Replies
5
Views
4K
  • Calculus and Beyond Homework Help
Replies
9
Views
2K
Replies
1
Views
1K
  • Introductory Physics Homework Help
Replies
2
Views
936
  • Advanced Physics Homework Help
Replies
2
Views
3K
Back
Top